2
$\begingroup$

For a differentiable real-valued function on $\mathbb{R}^n$, denoting $\partial_i f$ for the $i$th partial derivative, we can define the functional $$ T_n(f) = \sum_{i=1}^n \frac{1}{1 + \log(\|\partial_i f\|_2/\|\partial_i f\|_1)} \|\partial_i f\|_2^2. $$ Above, $\|\cdot\|_p$ denotes the usual $L^p$ norm under the standard Gaussian law $\gamma_n$ on $\mathbb{R}^n$.

The $L^1-L^2$ inequality implies that there exists a constant $C > 0$ such that for each $n \geq 1$ and any suitably regular $f \colon \mathbb{R}^n \to \mathbb{R}$, it holds that $$ \mathrm{Var}_{\gamma_n}(f) \leq C \, T_n(f), $$ where the variance is taken again with respect to standard Gaussian law on $\mathbb{R}^n$. We take $C$ above to be the smallest such constant: $$ C = \sup_{n \geq 1} \sup_{f} \frac{\mathrm{Var}_{\gamma_n}{f}}{T_n(f)}. $$

By considering linear functionals, it is clear that $C \geq 1$. On the other hand it is known that $C \leq 4$ (see, e.g., Corollary 5 in [1]).

Is the value of $C$ known? (In particular, are there any examples where $\mathrm{Var}(f) > T_n(f)$?)

[1] https://arxiv.org/abs/1105.4533

$\endgroup$
5
  • $\begingroup$ I don't understand the supremum over the dimension $n$ in the main inequality. $\endgroup$ Commented Dec 5 at 23:13
  • $\begingroup$ The constant could in principle depend on $n$, right? I am asking for the smallest that holds uniformly in the dimension. $\endgroup$ Commented Dec 5 at 23:27
  • $\begingroup$ But what happens to the supremum over $f$ as $n$ changes ? I would rather fix $n$ and look for the optimal $C_{n}$. $\endgroup$ Commented Dec 5 at 23:48
  • $\begingroup$ We know that $C_n \in [1, 4]$ for all $n$, so my interest is primarily in determining what the largest it can be is. Of course, if we can even determine $C_n$ then this is even better. It all depends on what level of accuracy you hope for. $\endgroup$ Commented Dec 5 at 23:49
  • $\begingroup$ Remark: I believe that one can select $\lambda > 0$ such that $f_\lambda(x) = x + \lambda (x^2 -1)$ makes it such that $\mathrm{Var}(f_\lambda) > T(f_\lambda)$. $\endgroup$ Commented Dec 5 at 23:57

0

You must log in to answer this question.

Start asking to get answers

Find the answer to your question by asking.

Ask question

Explore related questions

See similar questions with these tags.